Calculation of Kinetic Energy in Rotation Motion

In summary: The MoI is the inertia about an axis through the center of mass.Yes, "m" stands for the total mass.The expression for MOI is not clear.Problem statement says three identical rods of length L. From your ##I_{\rm c.o.m.}## I gather each rod has a mass of mL.
  • #1
Amartya Sen
8
0

Homework Statement


Respected Physics Gurus/experts...!

I am confused in the application of Kinetic energy Expression, i.e, KE = (1/2)MVCM2+(1/2)ICMω2
I had been trying out this question actually(it's pretty simple though:-p)...---

"A rigid body is made of three identical thin rods each of length L fastened together in the form of letter H. The body is free to rotate about a fixed horizontal axis AB that passes through one of the legs of the 'H'. The body allowed to fall from rest in a position in which the plane of H is horizontal. What is the angular speed of the body, when the plane of H is vertical?

https://physicsforums-bernhardtmediall.netdna-ssl.com/data/attachments/80/80350-15ec1ccefce05dd0a160c18183367070.jpg

Homework Equations


KE = (1/2)MVCM2+(1/2)ICMω2[/B]
& Work Energy Theorem


The Attempt at a Solution


I tried by considering the whole H at it's centre of mass and the applied the formulae but the answer I got was incorrect. My friend told me that since the 'H' is only rotating, the (1/2)MVCM2 term could be neglected...Why is it so? The CM has some velocity afterwards... Although the correct answer can be obtained by following this approach
Any help would be appreciated..!
 

Attachments

  • Untitled.jpg
    Untitled.jpg
    5.6 KB · Views: 359
Last edited by a moderator:
Physics news on Phys.org
  • #2
Amartya Sen said:
My friend told me that since the 'H' is only rotating, the (1/2)MVCM2 term could be neglected...Why is it so?
You can treat the body as purely rotating, but if you do, make sure you use the rotational inertia about the axis of rotation, not the center of mass.
 
  • #3
I know that...But why it is so..? And what's wrong with the application of the original formula?
 
  • #4
Hello Amartya, :welcome:

If you look carefully at the definition of moment of inertia -- in this case about an axis not through the center of mass -- you can see that the kinetic energy of the center of mass is already accounted for. To check you can work it out for a simple point mass instead of for an extended object.

There is a theorem - the parallel axis theorem, which google -- that links the moment of inertia around an axis through the center of mass to the moment of inertia around a parallel axis NOT through the center of mass
 
  • #5
BvU said:
If you look carefully at the definition of moment of inertia -- in this case about an axis not through the center of mass -- you can see that the kinetic energy of the center of mass is already accounted for. To check you can work it out for a simple point mass instead of for an extended object.

There is a theorem - the parallel axis theorem, which google -- that links the moment of inertia around an axis through the center of mass to the moment of inertia around a parallel axis NOT through the center of mass

Thanks BvU for your help..! :smile::smile::smile:
In the first place, I do know the parallel+perpendicular axis theorems. :wink:

But the formula says that we have to use the MoI about the COM of the object, not about the axis about which it is rotating.
Saying this, I want to also add that it's unclear that if we do have to use MOI about COM, so which plane we have to use?? ( As in this case---I think its the axis about which the direction of original rotation remains changed)
Next, I have used the formula exactly as it is in other questions regarding which there are no complaints...

Man, I do feel sometimes dizzy regarding some concepts in rotational motion..(toughest chapter for me)
It makes me go :doh::doh::doh:...

Plz, lead me out of trouble..
 
  • #6
No problem. Show your calculations and someone will point out where things go awry -- if at all (sometimes book answers can be wrong too). In your case, friend is wrong (or misquoted...).
 
  • #7
Here's my solution anyway...

Considering the body to be centred at the mid point of the middle leg of the 'H', we have,

mgl/2 =mv2/2 + [(ml2/12 + ml2/4 + ml2/4)ω2]/2
which on simplifying and putting v=ωl/2 gives:
ω=(12g/17l)^0.5

Totally wrong.
 
  • #8
Amartya Sen said:
But the formula says that we have to use the MoI about the COM of the object, not about the axis about which it is rotating.
But you don't have to use that formula. (But you can if you wish.)

Amartya Sen said:
mgl/2 =mv2/2 + [(ml2/12 + ml2/4 + ml2/4)ω2]/2
Does "m" stand for the total mass? Your expression for MOI is not clear.
 
  • #9
Problem statement says three identical rods of length L. From your ##I_{\rm c.o.m.}## I gather each rod has mass m. In that case there should be a 3 mgL/2 = 3 mv2/2 + ... on the left ! :smile:

And you can quickly check this parallel axis theorem as well.
 
  • #10
Oh Yeah..Yeah...I'm sorry. It should be as BvU said. Thanks BvU..
Anyway, even after the correction, answer comes to be wrong...
 
  • #11
How can you tell that the answer is wrong ?
 
  • #12
Yahoo...Finally I arrived the answer...It turns out that I had been replacing v2 by ω2l2 instead of ω2l2/4 in the final step ...?:)?:)?:)

Thanks pals... And sorry for wasting your precious time..if I did.
 

1. How is kinetic energy calculated in rotation motion?

In rotation motion, kinetic energy is calculated by multiplying half of the moment of inertia of the rotating object by the square of its angular velocity.

2. What is the moment of inertia in rotational motion?

Moment of inertia is a measure of an object's resistance to changes in rotational motion. It is calculated by summing up the product of the mass of each particle in the object and the square of its distance from the axis of rotation.

3. How does angular velocity affect the kinetic energy in rotation motion?

As the square of the angular velocity is used in the calculation of kinetic energy, an increase in angular velocity will result in a larger amount of kinetic energy. Similarly, a decrease in angular velocity will result in a decrease in kinetic energy.

4. Can the kinetic energy in rotation motion be negative?

No, the kinetic energy in rotation motion cannot be negative. This is because kinetic energy is a measure of the energy an object has due to its motion, and it cannot have a negative value.

5. Are there any assumptions made in the calculation of kinetic energy in rotation motion?

Yes, the calculation of kinetic energy in rotation motion assumes that the object is rigid and that the axis of rotation is fixed. It also assumes that there is no external torque acting on the object.

Similar threads

  • Introductory Physics Homework Help
Replies
21
Views
1K
Replies
7
Views
278
  • Introductory Physics Homework Help
Replies
33
Views
953
  • Introductory Physics Homework Help
Replies
9
Views
1K
  • Introductory Physics Homework Help
Replies
4
Views
1K
  • Introductory Physics Homework Help
Replies
20
Views
2K
  • Introductory Physics Homework Help
Replies
32
Views
2K
  • Introductory Physics Homework Help
Replies
15
Views
2K
  • Introductory Physics Homework Help
Replies
17
Views
722
  • Introductory Physics Homework Help
2
Replies
55
Views
2K
Back
Top